2022 AMC 10A Problems/Problem 11

Revision as of 02:10, 12 November 2022 by Kingravi (talk | contribs) (Solution)

Problem

Ted mistakenly wrote $2^m\cdot\sqrt{\frac{1}{4096}}$ as $2\cdot\sqrt[m]{\frac{1}{4096}}.$ What is the sum of all real numbers $m$ for which these two expressions have the same value?

$\textbf{(A) } 5 \qquad \textbf{(B) } 6 \qquad \textbf{(C) } 7 \qquad \textbf{(D) } 8 \qquad \textbf{(E) } 9$

Solution

We are given that \[2^m\cdot\sqrt{\frac{1}{4096}} = 2\cdot\sqrt[m]{\frac{1}{4096}}.\] Converting everything into powers of $2,$ we have \begin{align*} 2^m\cdot(2^{-12})^{\frac12} &= 2\cdot (2^{-12})^{\frac1m} \\ 2^{m-6} &= 2^{1-\frac{12}{m}} \\ m-6 &= 1-\frac{12}{m}. \end{align*} We multiply both sides by $m$, then rearrange and factor as \[(m-3)(m-4)=0.\] Therefore, we have $m=3$ or $m=4.$ The sum of such values of $m$ is $3+4=\boxed{\textbf{(C) } 7}.$

~MRENTHUSIASM

Alternatively, once we reach $m-6 = 1-\frac{12}{m}, we rearrange to get$m-7+\frac{12}{m}=0$. Multiplying both sides by$m$, we have$m^2-7m+12=0$. Since were asked to find the sum of all possible values of$m$, we use vieta’s formula to get the sum of the roots is$7=\boxed{C}$ ~KingRavi

See Also

2022 AMC 10A (ProblemsAnswer KeyResources)
Preceded by
Problem 10
Followed by
Problem 12
1 2 3 4 5 6 7 8 9 10 11 12 13 14 15 16 17 18 19 20 21 22 23 24 25
All AMC 10 Problems and Solutions

The problems on this page are copyrighted by the Mathematical Association of America's American Mathematics Competitions. AMC logo.png